Difference between revisions of "2013 AMC 12A Problems/Problem 17"

m (Solution)
(add solution 2)
Line 6: Line 6:
  
 
==Solution==
 
==Solution==
 
+
===Solution 1===
 
The first pirate takes <math>\frac{1}{12}</math> of the <math>x</math> coins, leaving <math>\frac{11}{12} x</math>.
 
The first pirate takes <math>\frac{1}{12}</math> of the <math>x</math> coins, leaving <math>\frac{11}{12} x</math>.
  
Line 26: Line 26:
  
 
We know there were just enough coins to cancel out the denominator in the fraction. So, at minimum, <math>x</math> is the denominator, leaving <math>1925</math> coins for the twelfth pirate.
 
We know there were just enough coins to cancel out the denominator in the fraction. So, at minimum, <math>x</math> is the denominator, leaving <math>1925</math> coins for the twelfth pirate.
 +
 +
===Solution 2===
 +
The answer cannot be an even number.
 +
 +
Consider the prime factorization of the starting number of coins. This number will be repeatedly multiplied by <math>\frac{n}{12}</math>. At every step, we are only removing twos from the prime factorization, never adding them (except in a single case, when we multiply by <math>\frac{2}{3}</math> for pirate 4, but that 2 is immediately removed again in the next step).
 +
 +
Therefore, if the 12th pirate's coin total were even, this can't be the smallest possible value; we can cut the initial pot in half and safely cut all the intermediate totals in half. So this number must be odd.
 +
 +
Only one of the choices given is odd, <math>1925</math>.
  
 
== See also ==
 
== See also ==
 
{{AMC12 box|year=2013|ab=A|num-b=16|num-a=18}}
 
{{AMC12 box|year=2013|ab=A|num-b=16|num-a=18}}
 
{{MAA Notice}}
 
{{MAA Notice}}

Revision as of 15:19, 25 January 2018

Problem 17

A group of $12$ pirates agree to divide a treasure chest of gold coins among themselves as follows. The $k^\text{th}$ pirate to take a share takes $\frac{k}{12}$ of the coins that remain in the chest. The number of coins initially in the chest is the smallest number for which this arrangement will allow each pirate to receive a positive whole number of coins. How many coins does the $12^{\text{th}}$ pirate receive?

$\textbf{(A)} \ 720 \qquad  \textbf{(B)} \ 1296 \qquad  \textbf{(C)} \ 1728 \qquad  \textbf{(D)} \ 1925 \qquad  \textbf{(E)} \ 3850$

Solution

Solution 1

The first pirate takes $\frac{1}{12}$ of the $x$ coins, leaving $\frac{11}{12} x$.

The second pirate takes $\frac{2}{12}$ of the remaining coins, leaving $\frac{10}{12}*\frac{11}{12}*x$.

Note that

$12^{11} = (2^2 * 3)^{11} = 2^{22} * 3^{11}$

$11! = 11 * 10 * 9 * 8 * 7 * 6 * 5 * 4 * 3 * 2$


All the 2s and 3s cancel out of $11!$, leaving

$11 * 5 * 7 * 5 = 1925$

in the numerator.


We know there were just enough coins to cancel out the denominator in the fraction. So, at minimum, $x$ is the denominator, leaving $1925$ coins for the twelfth pirate.

Solution 2

The answer cannot be an even number.

Consider the prime factorization of the starting number of coins. This number will be repeatedly multiplied by $\frac{n}{12}$. At every step, we are only removing twos from the prime factorization, never adding them (except in a single case, when we multiply by $\frac{2}{3}$ for pirate 4, but that 2 is immediately removed again in the next step).

Therefore, if the 12th pirate's coin total were even, this can't be the smallest possible value; we can cut the initial pot in half and safely cut all the intermediate totals in half. So this number must be odd.

Only one of the choices given is odd, $1925$.

See also

2013 AMC 12A (ProblemsAnswer KeyResources)
Preceded by
Problem 16
Followed by
Problem 18
1 2 3 4 5 6 7 8 9 10 11 12 13 14 15 16 17 18 19 20 21 22 23 24 25
All AMC 12 Problems and Solutions

The problems on this page are copyrighted by the Mathematical Association of America's American Mathematics Competitions. AMC logo.png